Difference between revisions of "2024 AIME II Problems/Problem 12"
Snowygalaxy (talk | contribs) |
|||
Line 1: | Line 1: | ||
− | : | + | Let <math>O(0,0),A(\tfrac{1}{2},0),</math> and <math>B(0,\tfrac{\sqrt{3}}{2})</math> be points in the coordinate plane. Let <math>\mathcal{F}</math> be the family of segments <math>\overline{PQ}</math> of unit length lying in the first quadrant with <math>P</math> on the <math>x</math>-axis and <math>Q</math> on the <math>y</math>-axis. There is a unique point <math>C</math> on <math>\overline{AB},</math> distinct from <math>A</math> and <math>B,</math> that does not belong to any segment from <math>\mathcal{F}</math> other than <math>\overline{AB}</math>. Then <math>OC^2=\tfrac{p}{q}</math>, where <math>p</math> and <math>q</math> are relatively prime positive integers. Find <math>p+q</math>. |
+ | |||
+ | ==Solution 1== | ||
+ | By Furaken | ||
+ | <asy> | ||
+ | pair O=(0,0); | ||
+ | pair X=(1,0); | ||
+ | pair Y=(0,1); | ||
+ | pair A=(0.5,0); pair B=(0,sin(pi/3)); | ||
+ | dot(O); | ||
+ | dot(X); | ||
+ | dot(Y); dot(A); dot(B); | ||
+ | draw(X--O--Y); | ||
+ | draw(A--B); | ||
+ | label("$B'$", B, W); | ||
+ | label("$A'$", A, S); | ||
+ | label("$O$", O, SW); | ||
+ | pair C=(1/8,3*sqrt(3)/8); | ||
+ | dot(C); | ||
+ | pair D=(1/8,0); dot(D); | ||
+ | pair E=(0,3*sqrt(3)/8); dot(E); | ||
+ | label("$C$", C, NE); label("$D$", D, S); label("$E$", E, W); | ||
+ | draw(D--C--E); | ||
+ | </asy> | ||
+ | |||
+ | Let <math>C = (\tfrac18,\tfrac{3\sqrt3}8)</math>. Draw a line through <math>C</math> intersecting the <math>x</math>-axis at <math>A'</math> and the <math>y</math>-axis at <math>B'</math>. We shall show that <math>A'B' \ge 1</math>, and that equality only holds when <math>A'=A</math> and <math>B'=B</math>. | ||
+ | |||
+ | Let <math>\theta = \angle OA'C</math>. Draw <math>CD</math> perpendicular to the <math>x</math>-axis and <math>CE</math> perpendicular to the <math>y</math>-axis as shown in the diagram. Then | ||
+ | <cmath>8A'B' = 8CA' + 8CB' = \frac{3\sqrt3}{\sin\theta} + \frac{1}{\cos\theta}</cmath> | ||
+ | By some inequality (i forgor its name), | ||
+ | <cmath>\left(\frac{3\sqrt3}{\sin\theta} + \frac{1}{\cos\theta}\right) \cdot \left(\frac{3\sqrt3}{\sin\theta} + \frac{1}{\cos\theta}\right) \cdot (\sin^2\theta + \cos^2\theta) \ge (3+1)^3 = 64</cmath> | ||
+ | We know that <math>\sin^2\theta + \cos^2\theta = 1</math>. Thus <math>\tfrac{3\sqrt3}{\sin\theta} + \tfrac{1}{\cos\theta} \ge 8</math>. Equality holds if and only if | ||
+ | <cmath>\frac{3\sqrt3}{\sin\theta} : \frac{1}{\cos\theta} = \frac{3\sqrt3}{\sin\theta} : \frac{1}{\cos\theta} = \sin^2\theta : \cos^2\theta</cmath> | ||
+ | which occurs when <math>\theta=\tfrac\pi3</math>. Guess what, <math>\angle OAB</math> ''happens'' to be <math>\tfrac\pi3</math>, thus <math>A'=A</math> and <math>B'=B</math>. Thus, <math>AB</math> is the only segment in <math>\mathcal{F}</math> that passes through <math>C</math>. Finally, we calculate <math>OC^2 = \tfrac1{64} + \tfrac{27}{64} = \tfrac7{16}</math>, and the answer is <math>\boxed{023}</math>. |
Revision as of 05:07, 9 February 2024
Let and be points in the coordinate plane. Let be the family of segments of unit length lying in the first quadrant with on the -axis and on the -axis. There is a unique point on distinct from and that does not belong to any segment from other than . Then , where and are relatively prime positive integers. Find .
Solution 1
By Furaken
Let . Draw a line through intersecting the -axis at and the -axis at . We shall show that , and that equality only holds when and .
Let . Draw perpendicular to the -axis and perpendicular to the -axis as shown in the diagram. Then By some inequality (i forgor its name), We know that . Thus . Equality holds if and only if which occurs when . Guess what, happens to be , thus and . Thus, is the only segment in that passes through . Finally, we calculate , and the answer is .